Quantcast
  • Register
PhysicsOverflow is a next-generation academic platform for physicists and astronomers, including a community peer review system and a postgraduate-level discussion forum analogous to MathOverflow.

Welcome to PhysicsOverflow! PhysicsOverflow is an open platform for community peer review and graduate-level Physics discussion.

Please help promote PhysicsOverflow ads elsewhere if you like it.

News

PO is now at the Physics Department of Bielefeld University!

New printer friendly PO pages!

Migration to Bielefeld University was successful!

Please vote for this year's PhysicsOverflow ads!

Please do help out in categorising submissions. Submit a paper to PhysicsOverflow!

... see more

Tools for paper authors

Submit paper
Claim Paper Authorship

Tools for SE users

Search User
Reclaim SE Account
Request Account Merger
Nativise imported posts
Claim post (deleted users)
Import SE post

Users whose questions have been imported from Physics Stack Exchange, Theoretical Physics Stack Exchange, or any other Stack Exchange site are kindly requested to reclaim their account and not to register as a new user.

Public \(\beta\) tools

Report a bug with a feature
Request a new functionality
404 page design
Send feedback

Attributions

(propose a free ad)

Site Statistics

205 submissions , 163 unreviewed
5,047 questions , 2,200 unanswered
5,345 answers , 22,709 comments
1,470 users with positive rep
816 active unimported users
More ...

  Making sense of the canonical anti-commutation relations for Dirac spinors

+ 3 like - 0 dislike
3311 views

When doing scalar QFT one typically imposes the famous 'canonical commutation relations' on the field and canonical momentum: $$[\phi(\vec x),\pi(\vec y)]=i\delta^3 (\vec x-\vec y)$$ at equal times ($x^0=y^0$). It is easy (though tedious) to check that this implies a commutation relation for the creation/annihilation operators $$[a(\vec k),a^\dagger(\vec k')]=(2\pi)^32\omega\delta^3(\vec k-\vec k') $$

When considering the Dirac (spinor) field, it is usual (see e.g. page 107 of Tong's notes or Peskin & Schroeder's book) to proceed analogously (replacing commutators with anticommutators, of course). We postulate \begin{equation} \{\Psi(\vec x),\Psi^\dagger(\vec y)\}=i\delta^3(\vec x -\vec y) \end{equation} and, from them, derive the usual relations for the creation/annihilation operators.

I'd always accepted this and believed the calculations presented in the above-mentioned sources, but I suddenly find myself in doubt: Do these relations even make any sense for the Dirac field? Since $\Psi$ is a 4-component spinor, I don't really see how one can possibly make sense out of the above equation: Isn't $\Psi\Psi^\dagger$ a $4\times 4$ matrix, while $\Psi^\dagger\Psi$ is a number?! Do we have to to the computation (spinor-)component by component? If this is the case, then I think I see some difficulties (in the usual computations one needs an identity which depends on the 4-spinors actually being 4-spinors). Are these avoided somehow? A detailed explanation would be much appreciated.

As a follow-up, consider the following: One usually encounters terms like this in the calculation: $$u^\dagger \dots a a^\dagger\dots u- u\dots a^\dagger a \dots u^\dagger $$ Even if one accepts that an equation like $\{\Psi,\Psi^\dagger\}$ makes sense, most sources simply 'pull the $u,\ u^\dagger$ out of the commutators' to get (anti)commutators of only the creation/annihilation operators. How is this justified?

EDIT: I have just realized that the correct commutation relation perhaps substitutes $\Psi^\dagger$ with $\bar \Psi$ (this may circumvent any issue that arises in a componentwise calculation). Please feel free to use either in an answer.

This post imported from StackExchange Physics at 2014-12-06 00:44 (UTC), posted by SE-user Danu
asked Dec 2, 2014 in Theoretical Physics by Danu (175 points) [ no revision ]
retagged Dec 5, 2014
Most voted comments show all comments
@glance I think you should be given the honors, and just include a reference to P&K substantiate your comment. Agreed that 3.89 is inappropriate.

This post imported from StackExchange Physics at 2014-12-06 00:45 (UTC), posted by SE-user joshphysics
@glance deriving the CCR for the creation/annihilation operators of the Dirac field would be more than satisfactory.

This post imported from StackExchange Physics at 2014-12-06 00:45 (UTC), posted by SE-user Danu
@joshphysics Of course, 3.89 is not the calculation for $\{\cdot,\cdot\}$ but it's the only explicit calculation, so that's why I was looking there ;)

This post imported from StackExchange Physics at 2014-12-06 00:45 (UTC), posted by SE-user Danu
Pretty anticlimactic, duh.

This post imported from StackExchange Physics at 2014-12-06 00:45 (UTC), posted by SE-user Daniel
@Daniel lol, yeah... I guess that's how these things are bound to end.

This post imported from StackExchange Physics at 2014-12-06 00:45 (UTC), posted by SE-user Danu
Most recent comments show all comments
@joshphysics damnit, I was looking at 3.89 and didn't see any components, and freaked out. I guess that settles it...

This post imported from StackExchange Physics at 2014-12-06 00:45 (UTC), posted by SE-user Danu
Somebody should post that as an answer! (FWIW I came to the same conclusion from Tong's notes)

This post imported from StackExchange Physics at 2014-12-06 00:45 (UTC), posted by SE-user David Z

1 Answer

+ 1 like - 0 dislike

One usually starts from the CCR for the creation/annihilation operators and derives from there the commutation rules for the fields. However, one can start from either (see for example here about this). Suppose we want then to start from the equal-time anticommutation rules for a Dirac field $\psi_\alpha(x)$: $$ \tag{1} \{ \psi_\alpha(\textbf{x}), \psi_\beta^\dagger(\textbf{y}) \} = \delta_{\alpha \beta} \delta^3(\textbf{x}-\textbf{y}),$$ where $\psi_\alpha(x)$ has an expansion of the form $$ \tag{2} \psi_\alpha(x) = \int \frac{d^3 p} {(2\pi)^3 2E_\textbf{p}} \sum_s\left\{ c_s(p) [u_s(p)]_\alpha e^{-ipx} + d_s^\dagger(p) [v_s(p)]_\alpha e^{ipx} \right\}$$ or more concisely $$ \psi(x) = \int d\tilde{p} \left( c_p u_p e^{-ipx} + d_p^\dagger v_p e^{ipx} \right), $$

and we want to derive the CCR for the creation/annihilation operators: $$ \tag{3} \{ a_s(p), a_{s'}^\dagger(q) \} = (2\pi)^3 (2 E_p) \delta_{s s'}\delta^3(\textbf{p}-\textbf{q}).$$ To do this, we want to express $a_s(p)$ in terms of $\psi(x)$. We have: $$ \tag{4} a_s(\textbf{k}) = i \bar{u}_s(\textbf{k}) \int d^3 x \left[ e^{ikx} \partial_0 \psi(x) - \psi(x) \partial_0 e^{ikx} \right]\\ = i \bar{u}_s(\textbf{k}) \int d^3 x \,\, e^{ikx} \overset{\leftrightarrow}{\partial_0} \psi(x) $$ $$ \tag{5} a_s^\dagger (\textbf{k}) = -i \bar{u}_s(\textbf{k}) \int d^3 x \left[ e^{-ikx} \partial_0 \psi(x) - \psi(x) \partial_0 e^{-ikx} \right] \\ =-i \bar{u}_s(\textbf{k}) \int d^3 x \,\, e^{-ikx} \overset{\leftrightarrow}{\partial_0} \psi(x) $$ which you can verify by pulling the expansion (2) into (4) and (5). Note that these hold for any $x_0$ on the RHS.

Now you just have to insert in the anticommutator on the LHS of (3) these expressions and use (1) (I can expand a little on this calculation if you need it).

most sources simply 'pull the $u, u^\dagger$ out of the commutators' to get (anti)commutators of only the creation/annihilation operators. How is this justified?

There is a big difference between a polarization spinor $u$ and a creation/destruction operator $c,c^\dagger$.

For fixed polarization $s$ and momentum $\textbf{p}$, $u_s(\textbf{p})$ is a four-component spinor, meaning that $u_s(\textbf{p})_\alpha \in \mathbb{C}$ for each $\alpha=1,2,3,4$. Conversely, for fixed polarization $s$ and momentum $\textbf{p}$, $c_s(\textbf{p})$ is an operator in the Fock space. Not just a number, which makes meaningful wondering about (anti)commutators.

This post imported from StackExchange Physics at 2014-12-06 00:45 (UTC), posted by SE-user glance
answered Dec 2, 2014 by glance (65 points) [ no revision ]
By "double arrow" do you mean $\Longleftrightarrow$ or two right arrows ontop of each other?

This post imported from StackExchange Physics at 2014-12-06 00:45 (UTC), posted by SE-user Kyle Kanos
I mean the arrow which points on both sides.. the symbol used to indicate a derivative on the right minus a derivative on the left: in $ a \bar{\partial}_\mu b \equiv a \partial_\mu b - (\partial_\mu a) b$ the symbol that would normally be used instead of the bar in $\bar{\partial}$

This post imported from StackExchange Physics at 2014-12-06 00:45 (UTC), posted by SE-user glance
\overset{\leftrightarrow}{\partial} $\overset{\leftrightarrow}{\partial}$

This post imported from StackExchange Physics at 2014-12-06 00:45 (UTC), posted by SE-user Robin Ekman
A single lined arrow that points both ways is \leftrightarrow: $\leftrightarrow$. You can put that over by using \overset{up}{down}: $\overset{\leftrightarrow}{\partial_\mu}$.

This post imported from StackExchange Physics at 2014-12-06 00:45 (UTC), posted by SE-user Kyle Kanos

Your answer

Please use answers only to (at least partly) answer questions. To comment, discuss, or ask for clarification, leave a comment instead.
To mask links under text, please type your text, highlight it, and click the "link" button. You can then enter your link URL.
Please consult the FAQ for as to how to format your post.
This is the answer box; if you want to write a comment instead, please use the 'add comment' button.
Live preview (may slow down editor)   Preview
Your name to display (optional):
Privacy: Your email address will only be used for sending these notifications.
Anti-spam verification:
If you are a human please identify the position of the character covered by the symbol $\varnothing$ in the following word:
p$\hbar$y$\varnothing$icsOverflow
Then drag the red bullet below over the corresponding character of our banner. When you drop it there, the bullet changes to green (on slow internet connections after a few seconds).
Please complete the anti-spam verification




user contributions licensed under cc by-sa 3.0 with attribution required

Your rights
...